Download as pdf or txt
Download as pdf or txt
You are on page 1of 9

Practice Final Exam II

FM423
Asset Markets

Part B
Question 2. (25 points)
(a) (12 marks) Suppose that the current forward and European-call-option prices on nat-
ural gas in sterling per MMBtu are as follows:

Maturity Forward Price Call Price, Strike = 3


6 months 2.2 0.002
one year 2.6 0.030

We have a flat term-structure at 3% per six-month (i.e., $1 today is worth $(1+0.03)


in 6 months, and $(1 + 0.03)2 in a year from today).

(i) (6 marks) Consider a swap agreement where you will receive 1 million MMBtu’s
in six months and 2 million MMBtu’s in one year, all for a fixed price of P per
MMBtu paid at each delivery time. What is the value of P that is consistent with
no arbitrage?
Solution: The present value of the gas received must match the present value of
the payments made:

2.2(1 + 0.03)−1 + 2 ∗ 2.6(1 + 0.03)−2 = P [(1 + 0.03)−1 + 2(1 + 0.03)−2 ],

which yields P = 2.46.


(ii) (6 marks) You wish to enter a contract with the following terms:
∗ In six months you will receive 1 million MMBtu’s of natural gas and in one
year you will receive 2 million MMBtu’s.
∗ The per MMBtu price you will pay will be the spot price at delivery time as
long as the spot price is not more than $3, and you will pay $3 per MMBtu
otherwise.
∗ In addition, you will make a payment P per MMBtu at each delivery time.
There is no time-zero payment. What is the new value of P that is consistent
with no arbitrage?


c LSE FM423 Page 1 of 9
Solution: The contract is equivalent to buying natural gas in the spot market
and at the same time placing a $3-cap on the price to be paid by buying calls.
The total cost of these calls must be equal to the present value of the two equal
payments:

0.002 + 2 ∗ 0.030 = P [(1 + 0.03)−1 + 2(1 + 0.03)−2 ],

which yields P = 0.0217.

(b) (5 marks) You are analyzing the returns of the Janus fund and you compare them with
those of the Magellan fund. The Janus fund has an average return of 12%, a standard
deviation of 20%, and a beta of 1.3. The Magellan fund has an average return of 10%,
a standard deviation of 17%, and a beta of 1. The current risk-free interest rate is 2%
and the expected market return is 8%. The market has a standard deviation of 11%.
What are the Sharpe Ratios and the Treynor Ratios of the two mutual funds? What
can you conclude about these funds’ performances? Discuss under what circumstances
we want to use these performance measures.
Solution: The Sharpe ratios are: SRJanus = (0.12-0.02)/0.20 = 0.50; SRMagellan =
(0.10-0.02)/0.17 = 0.47. According to the Sharpe ratio measure, Janus appears to
outperform Magellan.
The Treynor ratios are: TRJanus = (0.12-0.02)/1.3 = 0.077; TRMagellan = (0.10-
0.02)/1 = 0.08. According to Treynor ratio, Magellan appears to outperform Janus.
If one invest all his/her wealth with either Janus or Magellan, the Sharpe ratio is
the more appropriate performance measure. This is because it accounts for the total
risk (systematic+idiosyncratic) in the investor’s portfolio. If Janus or Magellan is
just one of many funds that an investor is invested in, then the Treynor ratio is a
more appropriate measure since the investor can diversify the idiosyncratic risk in this
particular portfolio away and only care about the systematic risk in this investment.

(c) (5 marks) Using monthly return data over the sample period of January 1980 to De-
cember 2016, you find that a zero-beta long-short strategy that buys firms with positive
earnings surprises and short firms with negative earnings surprises yields positive av-
erage returns. What are the possible interpretations of your findings? List at least
three.
Solution: The positive alpha is not unequivocal evidence of a profitable opportunity.
There are several possibilities. One is that the results may not be statistically signif-
icant in sample. The second is that the results may be the result of data-mining and
thus not hold out of sample. The third is Fama’s joint hypothesis problem; the strategy
may load on other priced risk factors. For example, the strategy may deliver CAPM
alpha simply because it loads on Fama-French’s size and value factors. Fourth, the
strategy may deliver positive alpha with respect to the true model of market equilib-
rium, but you may not be able to exploit the “anomaly” because of limits to arbitrage,


c LSE FM423 Page 2 of 9
for example, transaction costs or noise trader risk. Finally, this could be a true alpha
opportunity that can be exploited.

(d) (3 marks) A week in which the stock market has negative returns is followed by a week
of high market volatility. Does this statement violate any forms of market efficiency?
Explain.
Solution: No, EMH says nothing about whether return volatility should or should not
be predictable.


c LSE FM423 Page 3 of 9
Question 3. (25 points)
(a) (6 points) Show that early exercise of an American call on a non-dividend paying stock
is never optimal in frictionless markets.
Solution: Using put-call parity, we have
K
C AM ≥ C EU = P + S −
(1 + rT )T
> P +S−K
> S − K.

Therefore, C AM ≥ S − K, which is the value of immediate exercise.


(b) (4 points) Discuss the notion of delta-hedging and its role in the derivation of the
Black-Scholes PDE (a short intuitive explanation will suffice here; do not derive the
PDE).
Solution: The delta (∆) of an option is defined as the rate of change of the option
price with respect to the price of the underlying asset. It is the slope of the curve that
relates the option price to the underlying asset price. Delta is closely related to the
Black-Scholes PDE. The Black-Scholes differential equation can be derived by setting
up a riskless portfolio consisting of a position in an option on a stock and a position in
the stock. Expressed in terms of ∆, the portfolio is 1 option and -∆ shares of the stock.
Thus options can be valued by setting up a delta-neutral position and arguing that
the return on the position should (instantaneously) be the risk-free interest rate. The
reason a riskless portfolio can be set up is that the stock price and the derivative price
are both affected by the same underlying source of uncertainty: stock price movements.
In any short period of time, the price of the derivative is perfectly correlated with the
price of the underlying stock. When an appropriate portfolio of the stock and the
derivative is established, the gain or loss from the stock position always offsets the
gain or loss from the derivative position so that the overall value of the portfolio at the
end of the short period of time is known with certainty.
(c) (5 marks) The sensitivity of the price of a European call option in the Black & Scholes
model to calendar time t (i.e., Theta) is given by the following formula:
 
 −r(T −t)  St σ
Θ = − re KΦ(d2 ) − √ φ(d1 ) ,
2 T −t
where Φ(d) (φ(d)) is the cumulative distribution (probability density) function of a
standard normal, K is the strike price, T the maturity date, and d1,2 depend on all
parameters.
Explain the meaning of each of the two terms in the corresponding square brackets.
What are the signs of the terms for a European call option. Comment on whether you
expect the same signs to hold for European put options.


c LSE FM423 Page 4 of 9
Solution: For European Calls both terms are negative. T − t appears either attached
to r or to σ. The first term is the one attached to r: the pure discount effect. The
smaller time to maturity, the less K is discounted, i.e., the more has to be paid to
get the stock, i.e., the less the call is worth. The second term is attached to σ, and
represents the volatility effect. The larger t given T , the smaller time to maturity, i.e.,
the lower the volatility of ST given St (under Q), the lower the upside and downside
potential, therefore the lower the value of the call. For European puts the volatility
effect is of the same sign, but the interest rate effect is opposite so Θ can be positive
(though it is usually negative).

(d) (10 points) Shares of LSE.com will sell for either 200 or 120 in three months, with
probabilities 0.67 and 0.33 respectively. A European call with an exercise price of 160
sells for 25 today; a European put option with the same exercise price sells for 7. Both
options mature in three months.

(i) (4 points) What is the price of a three-month zero-coupon bond with a face value
of 100?
Solution: Call payoff: 200 − 160 = 40 in the up state, 0 in the down state. Put
payoff: 0 in the up state, 160 − 120 = 40 in the down state. Notice that call +
put has a riskless payoff of 40. The current price of call + put = 25 + 7 = 32.
Hence, the price of the zero is 32/0.4 = 80.
(ii) (6 points) Calculate the current price of the stock
∗ (3 points) by using risk neutral probabilities;
Solution: From above, 1 + r = 100/80 = 1.25. Let q be the risk neutral
probability of an “up” move. Then, using the risk neutral valuation formula
40q
for the price of the call, we have 25 = 1.25 , q = 25/32. Therefore the stork
price is:

200q + 120(1 − q)
S =
1+r
= 0.8(200 ∗ 25/32 + 120 ∗ 7/32)
= 146.

∗ (3 points) by replicating the stock with a portfolio of the call and the put.
Solution: In order to replicate the stock with x1 units of the call and x2 units
of the put, we need to satisfy the following system of equations:

40x1 = 200
40x2 = 120,


c LSE FM423 Page 5 of 9
which gives us x1 = 5; x2 = 3. Therefore the stock price is

S = Cx1 + P x2
= (25 ∗ 5) + (7 ∗ 3)
= 146.


c LSE FM423 Page 6 of 9
Question 4. (25 points)

(i) (12 points) This part is about forward and swap contracts.

(a) (2 points) Consider a two-year forward contract delivering a two-year coupon


bond with a face value of $100 and coupon rate of 20%. More specifically, the
forward price F will be paid in year 2 while the bond payments occur in years 3
and 4. The term structure is given by r0,1 =3%, r0,2 =4%, r0,3 =4.5%, r0,4 =5%.
Find the forward price F .
Solution:
−F 20 120
PV = + + = 0,
(1 + r0,2 )2 (1 + r0,3 )3 (1 + r0,4 )4
20 120
F = (1 + r0,2 )2 [ 3
+
(1 + r0,3 ) (1 + r0,4 )4 ]
20 120
= (1 + 4%)2 [ 3
+ = 125.73.
(1 + 4.5%) (1 + 5%)4 ]

(b) (3 points) Suppose, one year later the term structure becomes r0,1 =2%, r0,2 =3%,
r0,3 =3.5%, r0,4 =4%. What is the new value of the contract with forward price as
in part (a)?
Solution:
−F 20 120
V = + 2
+
(1 + r0,1 ) (1 + r0,2 ) (1 + r0,3 )3
−125.73 20 120
= + 2
+ = 3.82.
(1 + 2%) (1 + 3%) (1 + 3.5%)3

(c) (2 points) Consider a 4-year interest rate swap that each year exchanges fixed
payments at rate k to the cash flows of a floating rate note with spot annual
interest rate rt−1,t . The term structure is given by r0,1 =3%, r0,2 =4%, r0,3 =4.5%,
r0,4 =5%. Find the fixed rate k.
Solution:
k k k (k + 1)
1 = + 2
+ 3
+
(1 + r0,1 ) (1 + r0,2 ) (1 + r0,3 ) (1 + r0,4 )k
k k k (k + 1)
1 = + 2
+ 3
+ .
(1 + 3%) (1 + 4%) (1 + 4.5%) (1 + 5%)4

Solving this, we get k =0.049.


(d) (3 points) Suppose, one year later the term structure becomes r0,1 =2%, r0,2 =3%,
r0,3 =3.5%, r0,4 =4%. The notional amount is $100. What is the value of the


c LSE FM423 Page 7 of 9
interest rate swap in part (c) at t =1 (after the first cash flow is exchanged) to
the party receiving the fixed leg of this contract?
Solution: At t =1 only three payments are left, the value of the floater is equal
to the face value of $100. Hence,

PV = VF IXED (k) − VF RN
100 ∗ 0.049 100 ∗ 0.049 100 ∗ 0.049 + 100
= + + − 100
1 + 0.02 (1 + 0.03)2 (1 + 0.035)3
= 4.036.

(e) (2 points) Prove covered-interest-parity (CIP). [Note: Make sure every step in
your proof is justified.]
Solution: We discussed three approaches to prove CIP. Any approach would suf-
fice. Say USD is the home currency, and GBP the foreign currency, spot exchange
rate is S. Start with 1 GBP, invest in the UK market to earn (1 + rU K )T . Alter-
natively, convert to $S, and invest in the US market to earn (1 + rU S )T ; at time
T
T , convert back to GBP using the forward price, so the final payoff is S∗(1+r
F
US)
.
Since both investments are risk free, they must offer the exact same return, thus
1+rU S T
F = S ∗ ( 1+r UK
) .

(ii) (13 points) Historically, the dividend-price (d/p) ratio of the overall stock market in
any year is able to forecast movements in the stock market over the next few years;
e.g., a low d/p ratio is typically followed by poor returns in the stock market.

(a) (2 points) The “rational finance” camp, i.e., those who believe that prices are set
by rational investors, argues that the return pattern is driven by investors chang-
ing their perception of future stock market risk. Explain how such a mechanism
could, in principle, generate the return pattern mentioned above.
Solution: Suppose people decide that the stock market is going to be riskier going
forward than it was before. Then they will pay less for stocks, pushing p/d ratios
down and d/p ratios up (more formally, they are going to set a higher discount
rate in the present-value formula, which leads to lower prices, and hence lower
p/d ratios). Since they think the stock market is riskier, average returns going
forward will be higher, to compensate them for the higher perceived risk. So high
d/p ratios are followed by high returns.
(b) (3 points) The “behavioural finance” camp, i.e., those who believe that prices
are set in part by irrational investors, argues that the return pattern mentioned
above is due to investors’ sometimes exhibiting irrational exuberance or irrational
pessimism. Explain how such a mechanism could, in principle, generate the return
pattern mentioned above.


c LSE FM423 Page 8 of 9
Solution: Suppose that investors become irrationally exuberant. This means that
they push prices up too high, leading to high p/d ratios, and hence low d/p
ratios. Since the stock market is too high, it will subsequently correct downwards,
bringing low returns. So low d/p ratios are followed by low returns.
(c) (2 points) Describe the distinctions among the three forms of market efficiency.
Which form of market efficiency is violated by the behavioural story in part (b)?
Solution: The three forms of market efficiency differ in terms of the information
set. The evidence violates the semi-strong form of market efficiency.
(d) (3 points) Suppose that the d/p ratio on the overall stock market falls, so that
we forecast low returns on the stock market going forward. If we believe the
”behavioural finance” explanation for the return pattern mentioned above, given
in part (b), should we now lower our allocation to stocks, raise it, or leave it
unchanged? Explain.
Solution: We should lower our allocation to the stock market. If we believe the
behavioural finance story, then a low d/p ratio signals an overvalued stock market
that is on the verge of a downward correction.
(e) (3 points) Suppose that you are interested in figuring out which of the two mech-
anisms in (a) and (b) is really driving the return pattern mentioned above. What
analysis might you do to figure this out, i.e., to decide in favour of one or the
other mechanism? [Hint: Based on the rational story, how would you expect the
market risk to vary over time?]
Solution: The rational finance view of the pattern is that years with high d/p
ratios are followed by high returns because high d/p ratios represent times of
higher risk. We could test this by checking whether years with high d/p ratios
really are associated with higher risk. For example, is the stock market more
volatile in those years, or is the implied volatility on S&P 500 options in those
years particularly high?


c LSE FM423 Page 9 of 9

You might also like